Menu Close

a-n-1-a-n-2-2a-n-2-a-1-3-Prove-that-1-a-n-2-3-10-




Question Number 203313 by CrispyXYZ last updated on 16/Jan/24
a_(n+1) =a_n ^2 +2a_n −2, a_1 =3.  Prove thatΣ (1/(a_n +2)) ≤ (3/(10)).
$${a}_{{n}+\mathrm{1}} ={a}_{{n}} ^{\mathrm{2}} +\mathrm{2}{a}_{{n}} −\mathrm{2},\:{a}_{\mathrm{1}} =\mathrm{3}. \\ $$$$\mathrm{Prove}\:\mathrm{that}\Sigma\:\frac{\mathrm{1}}{{a}_{{n}} +\mathrm{2}}\:\leqslant\:\frac{\mathrm{3}}{\mathrm{10}}. \\ $$
Answered by witcher3 last updated on 16/Jan/24
 { ((a_(n+1) =a_n ^2 +2a_n −2)),((a_1 =3)) :}  a_n ≥n+2  by recursion a_1 =3≥1+2 true  suppose ∀n≥1 a_n ≥n+2  a_(n+1) =f(a_n );f(x)=x^2 +2x−2 increse in [0,∞[  a_n ≥n+2>0⇒a_(n+1) =f(a_n )≥f(n+2)=(n+2)^2 +2(n+2)−2  =n^2 +6n+6>n+3  ⇒a_(n+1) >n+3 ⇒∀n∈N a_n ≥n+2  a_(n+1) =a_n ^2 +2a_n −2⇒a_(n+1) +2=a_n (a_n +2)  ⇒((a_(n+1) +2)/(a_n +2))=a_n ≥n+2  ⇒Π_(k=1) ^n ((a_(k+1) +2)/(a_k +2))≥Π_(k=1) ^n (k+2)  ⇒((a_(n+1) +2)/(a_1 +2))≥3.4....(n+2)=(((n+2)!)/2)  ⇒a_(n+1) +2≥(5/2)(n+2)!  a_n +2≥(5/2)(n+1)!;∀n∈N  ⇒(1/(a_n +2))≤(2/5).(1/((n+1)!))⇒Σ_(n≥1) (1/(a_n +2))≤(2/5)Σ_(n≥1) (1/((n+1)!))=(2/5)(Σ_(n≥0) (1/(n!))−2)  e=Σ_(n≥0) (1/(n!))  Σ_(n=1) ^∞ (1/(a_n +2))=(2/5)(e−2)<(3/(10))  proof⇔2(e−2)<(3/2)⇒e<2+(3/4)=2.75>e true
$$\begin{cases}{\mathrm{a}_{\mathrm{n}+\mathrm{1}} =\mathrm{a}_{\mathrm{n}} ^{\mathrm{2}} +\mathrm{2a}_{\mathrm{n}} −\mathrm{2}}\\{\mathrm{a}_{\mathrm{1}} =\mathrm{3}}\end{cases} \\ $$$$\mathrm{a}_{\mathrm{n}} \geqslant\mathrm{n}+\mathrm{2} \\ $$$$\mathrm{by}\:\mathrm{recursion}\:\mathrm{a}_{\mathrm{1}} =\mathrm{3}\geqslant\mathrm{1}+\mathrm{2}\:\mathrm{true} \\ $$$$\mathrm{suppose}\:\forall\mathrm{n}\geqslant\mathrm{1}\:\mathrm{a}_{\mathrm{n}} \geqslant\mathrm{n}+\mathrm{2} \\ $$$$\mathrm{a}_{\mathrm{n}+\mathrm{1}} =\mathrm{f}\left(\mathrm{a}_{\mathrm{n}} \right);\mathrm{f}\left(\mathrm{x}\right)=\mathrm{x}^{\mathrm{2}} +\mathrm{2x}−\mathrm{2}\:\mathrm{increse}\:\mathrm{in}\:\left[\mathrm{0},\infty\left[\right.\right. \\ $$$$\mathrm{a}_{\mathrm{n}} \geqslant\mathrm{n}+\mathrm{2}>\mathrm{0}\Rightarrow\mathrm{a}_{\mathrm{n}+\mathrm{1}} =\mathrm{f}\left(\mathrm{a}_{\mathrm{n}} \right)\geqslant\mathrm{f}\left(\mathrm{n}+\mathrm{2}\right)=\left(\mathrm{n}+\mathrm{2}\right)^{\mathrm{2}} +\mathrm{2}\left(\mathrm{n}+\mathrm{2}\right)−\mathrm{2} \\ $$$$=\mathrm{n}^{\mathrm{2}} +\mathrm{6n}+\mathrm{6}>\mathrm{n}+\mathrm{3} \\ $$$$\Rightarrow\mathrm{a}_{\mathrm{n}+\mathrm{1}} >\mathrm{n}+\mathrm{3}\:\Rightarrow\forall\mathrm{n}\in\mathbb{N}\:\mathrm{a}_{\mathrm{n}} \geqslant\mathrm{n}+\mathrm{2} \\ $$$$\mathrm{a}_{\mathrm{n}+\mathrm{1}} =\mathrm{a}_{\mathrm{n}} ^{\mathrm{2}} +\mathrm{2a}_{\mathrm{n}} −\mathrm{2}\Rightarrow\mathrm{a}_{\mathrm{n}+\mathrm{1}} +\mathrm{2}=\mathrm{a}_{\mathrm{n}} \left(\mathrm{a}_{\mathrm{n}} +\mathrm{2}\right) \\ $$$$\Rightarrow\frac{\mathrm{a}_{\mathrm{n}+\mathrm{1}} +\mathrm{2}}{\mathrm{a}_{\mathrm{n}} +\mathrm{2}}=\mathrm{a}_{\mathrm{n}} \geqslant\mathrm{n}+\mathrm{2} \\ $$$$\Rightarrow\underset{\mathrm{k}=\mathrm{1}} {\overset{\mathrm{n}} {\prod}}\frac{\mathrm{a}_{\mathrm{k}+\mathrm{1}} +\mathrm{2}}{\mathrm{a}_{\mathrm{k}} +\mathrm{2}}\geqslant\underset{\mathrm{k}=\mathrm{1}} {\overset{\mathrm{n}} {\prod}}\left(\mathrm{k}+\mathrm{2}\right) \\ $$$$\Rightarrow\frac{\mathrm{a}_{\mathrm{n}+\mathrm{1}} +\mathrm{2}}{\mathrm{a}_{\mathrm{1}} +\mathrm{2}}\geqslant\mathrm{3}.\mathrm{4}….\left(\mathrm{n}+\mathrm{2}\right)=\frac{\left(\mathrm{n}+\mathrm{2}\right)!}{\mathrm{2}} \\ $$$$\Rightarrow\mathrm{a}_{\mathrm{n}+\mathrm{1}} +\mathrm{2}\geqslant\frac{\mathrm{5}}{\mathrm{2}}\left(\mathrm{n}+\mathrm{2}\right)! \\ $$$$\mathrm{a}_{\mathrm{n}} +\mathrm{2}\geqslant\frac{\mathrm{5}}{\mathrm{2}}\left(\mathrm{n}+\mathrm{1}\right)!;\forall\mathrm{n}\in\mathbb{N} \\ $$$$\Rightarrow\frac{\mathrm{1}}{\mathrm{a}_{\mathrm{n}} +\mathrm{2}}\leqslant\frac{\mathrm{2}}{\mathrm{5}}.\frac{\mathrm{1}}{\left(\mathrm{n}+\mathrm{1}\right)!}\Rightarrow\underset{\mathrm{n}\geqslant\mathrm{1}} {\sum}\frac{\mathrm{1}}{\mathrm{a}_{\mathrm{n}} +\mathrm{2}}\leqslant\frac{\mathrm{2}}{\mathrm{5}}\underset{\mathrm{n}\geqslant\mathrm{1}} {\sum}\frac{\mathrm{1}}{\left(\mathrm{n}+\mathrm{1}\right)!}=\frac{\mathrm{2}}{\mathrm{5}}\left(\underset{\mathrm{n}\geqslant\mathrm{0}} {\sum}\frac{\mathrm{1}}{\mathrm{n}!}−\mathrm{2}\right) \\ $$$$\mathrm{e}=\underset{\mathrm{n}\geqslant\mathrm{0}} {\sum}\frac{\mathrm{1}}{\mathrm{n}!} \\ $$$$\underset{\mathrm{n}=\mathrm{1}} {\overset{\infty} {\sum}}\frac{\mathrm{1}}{\mathrm{a}_{\mathrm{n}} +\mathrm{2}}=\frac{\mathrm{2}}{\mathrm{5}}\left(\mathrm{e}−\mathrm{2}\right)<\frac{\mathrm{3}}{\mathrm{10}} \\ $$$$\mathrm{proof}\Leftrightarrow\mathrm{2}\left(\mathrm{e}−\mathrm{2}\right)<\frac{\mathrm{3}}{\mathrm{2}}\Rightarrow\mathrm{e}<\mathrm{2}+\frac{\mathrm{3}}{\mathrm{4}}=\mathrm{2}.\mathrm{75}>\mathrm{e}\:\mathrm{true} \\ $$$$ \\ $$

Leave a Reply

Your email address will not be published. Required fields are marked *